ChaseDream
搜索
返回列表 发新帖
楼主: rx_11
打印 上一主题 下一主题

OG12-14 无人问过~~

[复制链接]
11#
发表于 2011-2-13 17:51:56 | 只看该作者
我问个没人问过的A,A说Over the next ten years,题干里面是within five years,这个时间点不需要对应?

谢谢
12#
发表于 2011-2-13 18:49:55 | 只看该作者
只是说double the number of movie screens是5年之内。

10年那个是和profit有关。

不矛盾
13#
发表于 2011-7-24 01:17:57 | 只看该作者
Justifying assumption is a sufficient assumption, in constrast to a necessary assumption.  As such, the justifying assumption can directly lead to the conclusion in the stimulus.

A can accomplish that feat while B cannot since B needs an additional assumption - after the economy is revitalized, people will go to the cinema.  If people go to night clubs instead, the argument falls apart.
-- by 会员 sdcar2010 (2010/12/13 22:02:59)



COOL explanation!
14#
发表于 2011-7-24 20:36:41 | 只看该作者
我是这么认为的,原文说they are predicting solid profi ts both for themselves and for the established cinema operators.即要对investors和established cinema operators都要有好处。
但B的假设成立的话,As distinct from the existing cinemas, most of the cinemas being planned would be located in downtown areas,in hopes of stimulating an economic revitalization of those areas.
也就是说现在的established cinema operators主要是建在非downtown区域,那么新建的电影院在downtown促进了经济发展后,会把部分人流从郊区带到市内,从而对established cinema operators没有好处,也就不符合原文的逻辑了。
15#
 楼主| 发表于 2011-9-14 23:31:36 | 只看该作者
一年以后碰巧再看到这个题,觉得完全懂了··虽然现在已经不考GMAT了,但是还是想回答一下这个问题,当时这个问题真让我纠结··

其实问题的关键在于they are predicting (1)solid profits both for themselvesand (2)for the established cinema operators.

B只满足(1),而不满足(2)。

(2)这个地方是B为什么错的关键!!所以做CR时要仔细的读题,特别是问题和结论部分。
16#
发表于 2011-11-3 15:34:25 | 只看该作者
美国的downtown和老百姓住的地方要很远,而老百姓住的地方到downtown的高速路附近就有很多电影院,所以在downtown兴建了,老百姓也不一定乐意去,到了晚上downtown就像个空城一样
17#
发表于 2012-3-11 22:46:21 | 只看该作者
一年以后碰巧再看到这个题,觉得完全懂了··虽然现在已经不考GMAT了,但是还是想回答一下这个问题,当时这个问题真让我纠结··

其实问题的关键在于they are predicting (1)solid profits both for themselvesand (2)for the established cinema operators.

B只满足(1),而不满足(2)。

(2)这个地方是B为什么错的关键!!所以做CR时要仔细的读题,特别是问题和结论部分。
-- by 会员 rx_11 (2011/9/14 23:31:36)




这个解释最好,全都满足
18#
发表于 2012-3-11 23:20:13 | 只看该作者
This question is not a paradox type of question.
19#
发表于 2012-3-11 23:41:53 | 只看该作者
那让我再好好看一看。。理解错了了吗。。
继续做题!!
20#
发表于 2012-4-8 18:52:50 | 只看该作者
终于明白了,it raises the question
of whether theaters in a revitalized downtown would draw business away from theaters in other
locations, thus reducing the established cinema operators’ profi ts.
一年以后碰巧再看到这个题,觉得完全懂了··虽然现在已经不考GMAT了,但是还是想回答一下这个问题,当时这个问题真让我纠结··

其实问题的关键在于they are predicting (1)solid profits both for themselvesand (2)for the established cinema operators.

B只满足(1),而不满足(2)。

(2)这个地方是B为什么错的关键!!所以做CR时要仔细的读题,特别是问题和结论部分。
-- by 会员 rx_11 (2011/9/14 23:31:36)

您需要登录后才可以回帖 登录 | 立即注册

Mark一下! 看一下! 顶楼主! 感谢分享! 快速回复:

手机版|ChaseDream|GMT+8, 2024-11-10 14:06
京公网安备11010202008513号 京ICP证101109号 京ICP备12012021号

ChaseDream 论坛

© 2003-2023 ChaseDream.com. All Rights Reserved.

返回顶部